User Avatar
7Sage Tutor
Tutor
PrepTests ·
PT103.S1.Q13
User Avatar
7Sage Tutor
Wednesday, Apr 30

Student Question

I still don't understand completely why E is the right answer and what the difference is between D being wrong.

Tutor Answer

Since this is a Must Be False (MBF) question, let's first break down the stimulus into a series of conditional statements. This will help us know with certainty what cannot be true.

1. People <-some-> think that in every barrel of politicians there are only a few rotten ones

2. If deceit is a quality of rottenness --> author believes all effective politicians are rotten

3. To do job properly (i.e. effectively) --> politicians must be deceitful

4. Scrupulously honest about obeying the rules --> /effective politician

Having laid out these statements, we can now say what must be true under the following conditions: - a sufficient condition is triggered; 2 - a necessary conditions is negated (i.e. the contrapositive).

 

It is for this reason that, when answering these kinds of questions, it is helpful to apply those aforementioned condition directly to the answer choices. By this I mean: we can now examine if an answer choice triggers a sufficient condition, negates a necessary condition, or simply has no bearing on any of the above conditional statements.

 

Before moving on, it's important to note that a MBF question does not mean all the incorrect answer choices must be true. Instead, simply being possibly true makes them incorrect as well. This makes the MBF an extremely high standard to surpass: we must know with 100% certainty that they cannot possibly be true.

 

Let's now take a look at answer choices D and E. Starting with D, we see that it say's "some deceitful politicians are ineffective." Does this trigger any of the conditions from our above statements? To make this clear, let's rewrite those statements below:

1. People <-some-> think that in every barrel of politicians there are only a few rotten ones

2. If deceit is a quality of rottenness --> author believes all effective politicians are rotten

3. To do job properly (i.e. effectively) --> politicians must be deceitful

4. Scrupulously honest about obeying the rules --> /effective politician

There are two possible conditions we can draw from D: "deceitful politicians" and "ineffective." Looking at the above statements, we see that both conditions exist as only as necessary conditions. In turn, nothing is triggered by either: we don't know if a deceitful politicians necessarily is an ineffective one, and vice versa. D, therefore, could be true; we simply don't know. Since it could be true, it follows that it does not have to be false.

 

Unlike D, however, E does trigger a follow on condition (in this as, a sufficient condition is triggered): "some scrupulously honest politicians are effective." The bolded and italicized part of that statement directly triggers statement #4 from above:

 

4. Scrupulously honest about obeying the rules --> /effective politician

 

As this statement tells us, anyone who is scrupulously honest cannot be an effective politician. This directly contradicts E, which says' that some of these kinds of politicians are actually effective. In turn, E cannot be true.

PrepTests ·
PT156.S4.Q15
User Avatar
7Sage Tutor
Wednesday, Apr 30

Student Question

So the “historical records” in AC A = the data they study?

Even tho I got this one right, AC A seemed quite tempting because I first thought that it means “what scholar has written”.

Tutor Answer

To directly answer your question: yes, it is reasonable to infer that "historical records" = "data" studied by scholars. In the context of this answer choice, "historical records" is a catch all term representing the information available to scholars.

 

But more broadly speaking, I 100% agree that A is a very tempting answer choice: it seems to give us a reason why historians probably focus more on studying successful than unsuccessful firms. The issue with A, however, is that "specific factors" does not equal overall business performance. Said another way, one need not know the "specific factors" that led to a business failing to know that the business failed. The implication: even granting the truth of A, scholars would still able to recognize whether or not a past business was successful or unsuccessful. In turn, we still don't know why scholars would purposefully over sample, and thus overestimate, the number of past successful businesses.

 

I noticed that you also said AC A was tempting because you thought it meant "what scholars have written." If possible, could you clarify what you meant by this statement? For example, are you referring to a specific part of the stimulus? I only ask to ensure I am answering your question in the best way possible!

PrepTests ·
PT103.S1.Q21
User Avatar
7Sage Tutor
Wednesday, Apr 30

Student Question:

In terms of speed, comparing logic obviously doesn’t make sense here. I understand the conditionals here and solved this problem on my own. The question is. What’s the real strategy? One of your tutors mentioned comparing language and sentence structure rather than the actual flaw in the conditional reasoning. If you do that here, it works. I guess I’m asking this: is this a viable strategy for CondR/Flaw/Parallel (high-level like this one) questions?

Tutor Answer:

This is an excellent question. When talking about strategy for Parallel and Parallel Flaw, I think it's always good to lead with the fact that these questions do take a lot of time, and (at least in my opinion) that's sort of by design. Really high level test takers generally tend to blaze through the Main Conclusion / Argument Part / Method questions that dominate the first fifteen of a given LR section simply because they're routine. The back half of the test is designed to take more time to do right, and these questions show up much more frequently there.

So, how do we get them to take less time? I really like the advice you mentioned another tutor giving -- I think it's an excellent starting place. Compare the language and sentence structure, with the understanding that the language itself isn't dispositive. I think that will help you diagnose the arguments quickly, without necessarily focusing too much on small details.

I had a pretty thorough method when I would do these questions. With Parallel Flaw questions, I would always start by checking if there was conditional reasoning in the argument when I was reading it. Generally, if there's conditional reasoning, then the flaw is going to be something that you've seen before -- a sufficiency vs. necessity flaw, or something to that end. Once you have the flaw, it's a lot easier to find an argument with the same flaw in the answer choices. For me, Parallel Flaw was almost easier than Parallel Reasoning, because I knew that the argument I was dealing with had a problem. The focus could be on finding the problem, as opposed to trying to really nail down a complete method.

In short: I like the advice you've been given, and I think if properly applied, it can be an excellent strategy on high level Parallel Flaw questions. Start with the flaw, and go from there.

PrepTests ·
PT106.S4.P4.Q22
User Avatar
7Sage Tutor
Wednesday, Apr 30

Student Question:

Please re-explain question 22

Tutor Answer:

This is a tough question. Let's go through it together, working only with what the author says in the passage. What you want to avoid with any "implied" RC question is stretching -- you should be able to select a specific portion of the passage that provides direct support for your inference.

I'm going to start by talking about why each of the wrong answers are wrong.

AC (B) is wrong because the author doesn't explicitly say today’s creators are less talented. I am willing to grant that the author praises past creators’ devotion to storytelling, but that alone isn't enough to support the inference made in AC (B).

AC (C) is wrong because the passage says today's industry is more interdependent with mass media, not less. (“Today this arrangement has mushroomed into an intricately interdependent mass-media entertainment industry.")

AC (D) is wrong, but is certainly more difficult to eliminate. In my mind, the passage focuses instead on how publicity functions (i.e., it drives profit and shapes film content), not on its entertainment value.) Therefore, I don't think there's a comparison, which would be needed to select AC (D).

AC (E) is wrong, and is very difficult to eliminate (again, in my opinion). That's because I do think that the passage presents the star system as an early marketing strategy (..the public was interested in actors' private lives, and that information about actors could be used to promote their films”). The thing is, I think the author is trying to point to the focus on hype and engagement as the reason for the decline in quality.

That leaves us with the correct answer:

AC (A) is correct because it is directly supported by the information available in the passage. The author contrasts past and present Hollywood, emphasizing that in the 1920s, publicity sparked curiosity but was backed by films created by “talented writers, directors, and producers devoted to the art of storytelling.” Today, the industry often prioritizes promotion over storytelling, and films are increasingly designed to generate curiosity rather than to emotionally engage audiences.

PrepTests ·
PT114.S1.Q9
User Avatar
7Sage Tutor
Tuesday, Apr 29

Student Question

Why is E better than D? Is there anything in the stimulus or the questions that indicates that dinosaurs being vulnerable to fatal respiratory problems would better explain why the dinosaurs died than that many were killed in the actual strike and then more in the subsequent tidal waves?

Tutor Answer

Regarding choice (D), it doesn't explain how even the killing of many dinosaurs could have caused them to all go extinct. Remember that "many" doesn't imply "most," and it should be treated as meaning "some." However, even if all dinosaurs in the area were killed, that doesn't explain why dinosaurs in other areas went extinct too-- how did dinosaurs on another continent, for example, die off?

For (E), we know from the stimulus that an asteroid would have hurled debris into the atmosphere for several months, which is a much farther-reaching event than an impact and tidal waves. So if dinosaurs could get a fatal infection, this would have killed off dinosaurs from a much larger area, possibly globally, which explains how all the dinosaurs went extinct.

PrepTests ·
PT122.S4.Q19
User Avatar
7Sage Tutor
Tuesday, Apr 29

Student Question

I thought B would be right because of the ‘last refuge’ part in the stimulus. Could you help clarify why B is incorrect. I also thought A was supported, but b more strongly supported because of its specificity.

Tutor Answer

In clarifying why B is incorrect, I think it will be helpful to first start with a discussion of specificity and how it relates to Most Strongly Supported (MSS) questions.

 

Specificity can be an important component of MSS questions. If an answer choice specifically matches something that was said in the stimulus, then we know it is indeed the correct answer. In that sense, specificity can be useful in answering these types of questions.

 

But it is important to note that "specificity" is only useful in so far as it helps illuminate information that could be true. An answer choice (AC), for example, is not more likely to be true only because it makes a very specific statement. It is only more likely to be true if states information that we actually know to be true, or very likely to be true given what we read in the stimulus.

 

Which is why specific ACs can actually be dangerous options: if they are not exactly stated and/or matched in the stimulus, then they cannot be correct.

 

AC B is a prime example of this idea. AC B tells us the following: Unless resource managers regularly intervene in most of the world's remaining forests, many of the world's most endangered species will not surviveThis is a very specific statement, but does it match the stimulus?

 

The answer to this question is no, it does not. Looking at the second sentence of the stimulus tells us why: These fragmented forest ecosystems have typically lost their ability to sustain themselves in the long term, yet they include the last refuges for some of the world most endangered species. "Last refuges" is the key phrase in that sentence. It tells us that "last refuges" are in danger if no outside intervention is made. But importantly, it does not tell us that losing last refuges equates to extinction.

 

Think of it this way: Do we know zoo's won't take in these animals? Do we know new preserves won't be created? We don't know the answers to these questions, which means we can't say for certain if losing the "last refuges" for these animals will lead to their extinction.

 

AC B, therefore, was too specific. It described a specific consequence (extinction) that is not actually supported in the stimulus. Had it instead said something to the effect of "these endangered species would lose their last refuges," then it would indeed have been correct. But instead said something else, something that constrained the consequences of non-intervention in a way that was unsupported by the stimulus.

 

It is for this reason that I would avoid looking only for specificity in potential ACs. Instead, approach ACs with the mindset of "Is this actually supported?" Seen in this light, ACs which are highly qualified and which offer only a limited number of possibilities are the ones most likely to be true. AC A is a great example of this concept: it creates a scenario in which a certain sufficient condition has been triggered (i.e. if no one intervenes), it qualifies a number somewhere above 50% (i.e. most), and it says a consequence will result to some number above one (i.e. at least). Which is to say, AC A perfectly matches the stimulus by not going outside the limits of what could be true. It is this element of AC A that makes it the correct AC.

 

 

PrepTests ·
PT156.S1.P1.Q4
User Avatar
7Sage Tutor
Tuesday, Apr 29

Student Question

Q4) C vs E

*Para 1 says — As a possible solution, agricultural researchers, development experts, and policy makers are “searching for ways” to conserve the wealth of genetic information contained in crops known as heirloom varieties

*Last Para says — author argued “knowledge system” should be preserved

Doesn’t this mean that other people recognized the potential of heirloom varieties as a solution, and author proposed an actual way to conserve it?

I thought the structure of the passage is

1 - problem/potential thing (heirloom) that can serve as a solution

2 - why it’s potential

3 - how to solve the problem with that thing (conserve knowledge system) & why it’s urgent

I’m not saying C should be an answer, I just want to know what makes E a better answer. The more are the better!

Tutor Answer

I think that you're right about the general structure:

1. There's a problem identified: North American agriculture relies heavily on a narrow genetic base, making it vulnerable.

2. There's a potential solution introduced: Heirloom varieties developed by indigenous farmers could help.

3. Now, the author does some expansion: Explanation of why heirloom varieties are valuable — genetic diversity, pest resistance, environmental adaptation.

4. The author emphasizes urgency and makes a proposal: Immediate steps are needed to preserve indigenous knowledge systems alongside conserving heirloom seeds.

Now, focusing on the central portion of your request: Does the author propose a novel solution [AC (C)] or support a promising one [AC (E)]?

 

Here's the key, in my mind:

*"Propose a novel solution" (C) means introducing an idea that others have not yet considered (which feels like it would be required for something to be "truly" new).

*"Support a promising approach" (E) means endorsing and expanding on an approach that people already recognize as valuable, backing it with further reasoning.

The author does not claim to be introducing a brand-new solution. In fact, the very first paragraph says "agricultural researchers, development experts, and policy makers are searching for ways to conserve" heirloom varieties — so others already recognize heirloom crops as important.

 

The author's main action with this passage is:

Strengthening that approach by emphasizing that it's not enough to just save the seeds — you must also save the indigenous knowledge that makes them thrive.

Urging immediate action to support this broader conservation effort.

Basically, the "solution" (using heirloom varieties) was already under discussion by others. The author's contribution is supporting that solution and adding nuance (preserve the associated knowledge too).

Thus, AC (E) ("support a promising approach to a problem") fits better than AC (C).

PrepTests ·
PT109.S4.Q21
User Avatar
7Sage Tutor
Monday, Apr 28

Student Question

Wouldn’t the translation of the first sentence be BA ←-some→ FC? Since the second sentence is GA → FC, I thought that the error was that you can’t get a conclusion from an all then a some statement. Why is it a mistaken reversal? The chained conditional logic would be GA → FC ←-some→ BA. From this, you can’t draw a conclusion at all, so why would it be different if the GA and the BA are switched?

Tutor Answer

This is tricky, because the "some" language does make it tempting to create an "GA --> FC BA" diagram. But there is catch: the argument is not saying that all good actions encompass every favorable consequence. This, however, would be the implication of the aforementioned diagram.

Thinking of it in this manner might help: is every favorable consequence a product of a good action? No, not based on what we learned. We only were told that favorable consequences are a necessary condition of a good action.

For example, if I said "All cats live indoors, and some people live indoors," that would equate to "Cats --> indoors people." Would that mean that some cats are people? No, it would not, because the necessary condition does not imply a sufficient condition. In this question, therefore, we cannot say some good actions are bad actions.

PrepTests ·
PT144.S1.P4.Q23
User Avatar
7Sage Tutor
Monday, Apr 28

Student Question

Q23: Why is D-RADICAL a better choice than A and E. I do not see the difference between A, D and E. Q24: I do not understand the explanation of the video.

Tutor Answer

For Q23, D is correct because the author criticizes Popper for taking a real idea (the power of negative evidence) and pushing it too far by applying it in too extreme a way. Saying that negative evidence is all that matters and positive evidence has no value is a radical conclusion that goes beyond the basic logic. A is wrong because the problem isn’t that Popper applied the idea to a case it doesn’t fit—it’s that he overextended its importance everywhere. E is wrong because while exaggeration sounds close, the focus is not just about relevance to one theory; it’s about drawing an extreme conclusion across scientific reasoning more broadly. D captures that broader overreach better.

For Q24, the right answer is B because at the end of Passage A, the author says that when a prediction fails, there is rarely a simple, single disproof. In Passage B, when astronomers couldn’t predict Uranus’s orbit correctly, it didn’t automatically disprove Newton’s laws—they had multiple possible explanations, like problems with auxiliary assumptions. That matches exactly what the author of Passage A says about negative evidence not being conclusive. So B is the clearest match. But this is a tough question! Thank you for reaching out on that.

PrepTests ·
PT125.S2.Q17
User Avatar
7Sage Tutor
Monday, Apr 28

Student Question

Why B is not correct?

Tutor Answer

The issue with choice (B) is that, while it may be an illustration, it is not supporting the claim that “the criteria of legal responsibility for an action include but are not the same as those for moral responsibility.” The argument never made this claim. In fact, the argument makes the claim that legal responsibility is different than moral responsibility, but the argument never said that criteria for legal responsibility includes criteria for moral responsibility.

PrepTests ·
PT136.S2.Q12
User Avatar
7Sage Tutor
Monday, Apr 28

Student Question

It seems like this question relies on me knowing that moss + rainforests are major contributors to the amount of oxygen entering our atmosphere. Over time, I recognized that A did justify the argument, but not to the degree that B would. At the same time, unlike A, B required me to assume that the current rate of sphagnum moss depletion = significant reduction in the amount of oxygen entering the atmosphere. In fact, I wouldn’t have needed to make any assumptions with A. So, with that in mind, I picked A over B.

My question is - is there any way for me to get this answer correct without that outside knowledge? And, if you didn’t also have this outside knowledge and were down to A and B, how would you have justified accepting B’s assumptions?

Tutor Answer

I can see where you’re coming from for this problem. I would say that for this problem, it’s implied by the wording that “millions of acres” would be significant. We don’t want to make too many assumptions, but if it’s implied by the wording of the argument, it’s ok. The argument also wouldn’t really make sense if we were depleting something insignificant, and we are trying to make the argument work for this question type.

PrepTests ·
PT120.S1.Q24
User Avatar
7Sage Tutor
Monday, Apr 28

Student Question

Why is D not correct?

Tutor Answer

For this problem, we want to find an answer that goes against what the consumer advocate is saying. The consumer advocate says that instructions should make the assembly process easier than without instructions. Choice (D) says that most people understand the instructions, which doesn’t go against what the advocate says, so it can’t be the correct answer. If most people understand the instructions, that seems to support the idea that instructions should make the assembly process easier. Choice (C) says most people assemble things really easily without instructions, which weakens what the consumer advocate is saying about instructions, and therefore is the correct answer.

PrepTests ·
PT154.S2.Q21
User Avatar
7Sage Tutor
Monday, Apr 28

Student Question

I can see why the other ACs are wrong, but I am struggling to see why E is right.

Tutor Answer

Right off the bat, I think it's important to note that this is an objectively difficult question. Having some questions here is totally normal and expected.

 

To help cut through the complexity this question, let's first start with the answer choice itself. What is actually being said in E?

 

The first part of the sentence says: "If a weather pattern with a natural cause has a seven-day cycle..." Understanding this clause is crucial in understanding the answer choice. Why? Because this clause restates the exact same kind of weather cycle described in the stimulus.

 

Modifying the clause might help make this clear. By changing it to "If a weather pattern with a seven-day cycle has a natural cycle" we now see that the weather pattern being described is simply one with a seven day cycle. A limit is being applied to the scope of the cycle describe in this AC in order to make it the same as that described in the stimulus: The stimulus gave us a seven-day cycle, and this answer choice is now providing us with the same thing.

 

In effect, then, this part of the answer choice is saying, "if nature caused the type of weather pattern describe in the stimulus.."

 

All together, and based on our understanding of that first clause, the full AC is saying: "If nature caused the type of weather pattern described in the stimulus, then that cause occurred on a seven-day cycle."

 

Why does "seven day cycle" matter? Because that is the only type of weather pattern referenced in the stimulus. By doing so, the author limited natural causes to those that occur over a seven day time period. But herein lies an assumption: not all weather patterns necessarily occur over a seven-day time period. For example, what if a two-day cycle of weather created the type of weather pattern described in the stimulus (i.e. 7 days in total: five sunny, two cloudy)? If that was the case, then the fact that seven-day cycle's of weather have no appreciable effect on the kinds of weather pattern described the stimulus would be largely irrelevant. Other natural causes, those not referenced by the author, might be causing that weather pattern.

 

Which is why AC E is so crucial to the author. The statement "If nature caused the type of weather pattern described in the stimulus, then that cause occurred on a seven day cycle" precludes that very possibility. The causes of that weather pattern, in which five days of sun and two days of clouds occur, have now been limited to seven-day natural cyclesIn turn, since we already know that naturally occurring seven day cycle do not cause measurable weather patterns, it remains possible that only humans are too blame. It is for this reason that AC E is the correct answer.

PrepTests ·
PT105.S1.Q19
User Avatar
7Sage Tutor
Monday, Apr 28

Student Question

I understand answer choice D to be attempting to weaken the plausibility that life (plants/microbes) was causing the carbon 14 to be in the 1.2-billion-year-old rocks.

I thought of this alternative cause like this:

Carbon 14 in Atmosphere —cause—> Carbon 14 in rock.

But since plants and microbes release Carbon 14 into the atmosphere after they die, how come answer choice D would weaken the argument if Carbon 14 released into the atmosphere from life (plants/microbes) causes the Carbon 14 to be in the 1.2-billion-year-old rocks?

I thought of it like this:

Life (Plants/Microbes) —c—> Carbon 14 in Atmosphere —c—> Carbon 14 in 1.2 billion rock.

Tutor Answer

Answer choice (AC) D is an objectively tricky AC. It's very tempting, because it seems to provide evidence showing life was present in soil 1.2 billion years ago. If that was the case, then this AC would indeed strengthen the argument.

 

But that is not what is being said in D. Instead, D tells us that C14 entered the soil directly from the atmosphere, soil of which then formed into rocks. In turn, C14 present in those same rocks would not be indicative of life. Why? Because by directly entering the soil from the atmosphere, it is no longer the case that "plants" or "microbes" were necessarily present.

 

D is highlighting an easily overlooked assumption: C14 is only ever captured and released into the atmosphere by plants or microbes. But what if non living things could perform the same action? If so, the presence of C14 would not be indicative of life. AC D provides us with just that scenario.

PrepTests ·
PT117.S4.Q26
User Avatar
7Sage Tutor
Monday, Apr 28

Student Question

I understand this question more now that I’ve watched the video but I’m having trouble understanding the logic chains. Are there videos or lessons you can refer me to that go over the logic chains.

Tutor Answer

A solid foundation in logic, specifically formal logic of the kind used on the LSAT, is important in understanding how to diagram conditional statements (i.e. logical chains). For this, I highly recommend reviewing the 7Sage Core Curriculum. Here you will find the following sections:

*Conditional and Set Logic

*Logic of Intersecting Sets

*Formal Logic Flaws

I recommend completing each of these sections. But in addition, and to learn specifically how to create the kinds of diagrams used in this question, I recommend also reviewing both the "Psuedo-Sufficient Assumption" (PSA) and "Sufficient Assumption" (SA) sections as well. These sections will focus more on "filling in the gaps" of logic chains in order to find sufficient assumptions. That skill in particular is very important for both PSA and SA questions.

PrepTests ·
PT124.S3.Q19
User Avatar
7Sage Tutor
Monday, Apr 28

Student Question:

How does having Lind NOT guarantee Knight? There just has to be two max

Tutor Answer:

You’re right to notice that there can be up to two authors included, but the specific logic from the stimulus gives us a stronger relationship between Lind and Knight. From the last sentence, we know that if the book contains Knight, then it must also contain Jones, and it cannot contain Lind. That sets up two conditional rules: K → J and K → /L.

When you have a conditional statement like K → /L, and you know that the necessary condition (/L) is not true—meaning that L is present—you apply a rule of conditional logic: when the necessary condition fails, that forces the sufficient condition to fail too. In other words, if we have L, we must also have /K. So having Lind in the textbook guarantees that Knight cannot be there, because the failure of the necessary condition reflects back and forces the failure of the sufficient condition. That's how we get to the correct answer choice of A--which is our must be true response!

PrepTests ·
PT132.S2.Q19
User Avatar
7Sage Tutor
Monday, Apr 28

Student Question:

I do not see how E is wrong. Can’t the indy pet stores not sell fish and still sell birds?

Tutor Answer:

First, take a look at the diagrammed image of the stimulus below. To begin, just so we’re on the same page about the abbreviations from my diagram: PS stands for Pet Store (that is, any Pet Store in West Calverton), EB stands for Exotic Birds, TF stands for Tropical Fish, G stands for Gerbils, and Ind stands for Independently Owned.

Now for answer choice E—you're absolutely thinking along the right lines in noticing that an independently owned store might not sell tropical fish and still sell exotic birds. But the key here is that the question is asking what must be true, and E only could be true, not must. If you look closely at the second image, you’ll see that the rule for independent stores is: Ind → (/TF or EB). I got to this relationship by taking the contrapositive of these two conditions.

That means an independently owned pet store must either not sell tropical fish, or sell exotic birds. It’s an or relationship, and in logic, an "or" means that satisfying either condition is enough. So if we know an independent store does not sell tropical fish, the rule is already satisfied—we don't need to know anything further about whether they sell exotic birds. They might, or they might not.

Since the rule doesn’t guarantee that exotic birds will be sold in this case, E is not something that must always happen.

As for the correct answer, D, it's actually working off of the same rule that E is. Except in the case of D, we don't need to take the contrapositive of the conditional. D may be converted into formal logic as the following: TF + /EB --> /Ind. This is precisely what we have in our original diagram, and it must be true in all cases.

PrepTests ·
PT128.S3.Q14
User Avatar
7Sage Tutor
Edited Saturday, Sep 13

Student Question:

When will the answer choice C be correct in this sort of flaw question? Does the conlcusion need to specifically include the word only for it to assume there is no other sufficient?

Tutor Answer:

Let's start by discussing the question itself. Answer choice C is wrong here because the argument is not presuming that plagiarism is the only factor contributing to professional misconduct. Instead, the flaw is that it assumes a causal relationship just because there’s an association between plagiarism and later misconduct. The argument says that reducing plagiarism will reduce misconduct, but it never claims or even implies that plagiarism is the sole cause. Therefore, C is too strong for this flaw. The correct answer here is E because the argument infers the existence of a causal connection merely on the basis of a correlation/"association"—students who plagiarize are more likely to later engage in misconduct, but correlation alone isn’t enough to conclude causation.

Now, let me answer your original question. Answer choice C would be correct in a situation where the argument treats one factor as being the only significant cause of an outcome, without considering or ruling out other possible causes. Importantly, the conclusion does not need to literally use the word "only" for that flaw to be present. What matters is whether the argument’s structure acts as if there are no other possibilities. If the argument functionally assumes that one cause alone fully explains the outcome, while ignoring the possibility of other contributing factors, that’s when C would match.

You can see a similar flaw to that described in C in PT 117 Section 2 Question 25, which you can find here: https://7sage.com/discussion/2373/pt117-s2-q25-the-first-bicycle-the-draisienne-was

In that question, the correct answer is E: "ignores, without giving justification, alternative possible explanations of the initial failure of bicycles." There, the argument sees that bicycles initially failed to catch on and assumes a particular cause for that failure, without considering other possible explanations. In both cases, the argument improperly ignores other possible causes and treats a single observed factor as if it fully explains the outcome, even though alternative explanations could easily exist.

PrepTests ·
PT115.S1.P4.Q28
User Avatar
7Sage Tutor
Sunday, Apr 27

Student Question

For Q28, I don’t get why it’s E over B - I felt B was the better answer. For the debate described in the hypothetical, two different religions came to a different conclusion based on different accounts of the same event - both of those accounts were based on different religious texts. So it seems like the same type of evidence (religious texts) but different evidence supporting each view. I thought B was most analogous because two different eye witness accounts of the same event are of the same type (eye witness account) but with two different versions of the same event. I feel like E would be analogous to the passage if, instead of the two religions disagreeing bc of two different religious texts, one’s view of universe’s creation is based on a religious text the other based on oral retellings of the universe creation, because those are two diff types of evidence.

Tutor Answer

We're asked to pick the answer that is most analogous to the hypothetical debate that is given to us in the fourth paragraph of the passage. AC (B) provides a possibly analogous type of debate.

AC (B): A debate among jurors attempting to determine which of two conflicting eyewitness accounts of an event is to be believed.

Okay, in this debate, the jurors are debating whether to believe one account or another. Is that analogous to the way that the "adherents of different religions" treat the evidence that they are dealt with?

Line 43: "While each religion may be confident that its cosmology is firmly grounded in its respective sacred text, there is little hope that conflicts between their competing cosmologies could be resolved by recourse to the texts alone."

Here's our problem. The analogy tells us that the jurors are debating what to believe about an event -- and while that fits, the way that they treat the evidence doesn't. The jurors are debating what account to believe, but we have no reason to believe that both sides of the jury room are firmly rooted in their own interpretation. It could be an open, free-flowing debate. We know that the adherents of different religions in the example have come to different conclusions, because we're told that each religion is "confident that its cosmology is firmly grounded in its respective sacred text."

That's why AC (E) is a better choice here. AC (E) starts with the fact that the historians have already drawn different conclusions about the same event. We know that's already happened in this debate -- but we don't know anything about the debate in the hypothetical jury room.

PrepTests ·
PT111.S1.Q17
User Avatar
7Sage Tutor
Sunday, Apr 27

Student Question

In the correct answer choice (E), I’m hung up on the following verbiage: “An inconsistency that, as presented, has MORE THAN ONE POSSIBLE RESOLUTION etc etc.” Even after viewing review materials for this question, I still feel that the inconsistency is only presented as having ONE resolution: the author does not propose any alternative resolutions to his claim that Antarctica’s ice sheet was at one point melted. The author does not supply any other reasons for the presence of the aforementioned fossils. Bearing this in mind, why does the correct answer choice state that the inconsistency in the argument is presented as having more than one possible resolution?

Tutor Answer

I think this comes down to how you interpret the sentence. The author only presents/supplies one resolution, but the inconsistency is presented in such a way that, in reality, it has multiple possible resolutions. The author didn't present the inconsistency in a way that precluded all other resolutions, they just ignored them.

If the answer choice instead said "An inconsistency that is presented as having more than one possible solution is treated as though only one resolution is possible" then I think that would be more in line with your current interpretation.

PrepTests ·
PT107.S1.Q22
User Avatar
7Sage Tutor
Sunday, Apr 27

Student Question

Is there a simpler way to understand this? It doesn't make any sense to me, I'm getting really lost in the stimulus.

Tutor Answer

When I encounter a long stimulus like this, I consider taking bullet point notes to help organize key facts. Here's what I came up with:

-Government says nuclear power plant safe, don't fear nuclear accidents

-Government says limited financial liability is okay because industry needs to be protected from bankruptcy

-Government says unlimited financial liability causes bankruptcy only if injury claim can be sustained (conditional!)

-Government says sustained only if injury results from nuclear accident (conditional!)

-CONCLUSION: fear nuclear accidents

Basically, the author is making the argument that because the government acted to prevent bankruptcy resulting from nuclear accidents, we should actually fear nuclear accidents.

But the government said nuclear accidents won't happen. Why try to prevent something that won't happen? That would be irrational, right? Well, our argument would be helped if we assume that the government isn't being irrational here. If we assume that the government will act to prevent a situation only if that situation is possible, then we have proof that nuclear accidents are possible and that the government was not being very forthcoming about that possibility-- so answer (D) works!

PrepTests ·
PT129.S2.Q25
User Avatar
7Sage Tutor
Sunday, Apr 27

Student Question

Can you explain the piece about when the sufficient is failed, the rule no longer applies? Usually, I take a failed sufficient condition to mean that we don’t know what happens, but here this means that Brimley doesn’t have to register. I know that this is the only law, but does that really mean he doesn’t have to register? Should the diagram be a biconditional?

Tutor Answer

With MBT questions, I generally like to simplify the conditionals I'm working with -- especially since these questions often feature a key inference early on that I might miss if I'm not looking out it. Here's the stimulus in this question, for reference:

Stimulus: The law of the city of Weston regarding contributions to mayoral campaigns is as follows: all contributions to these campaigns in excess of $100 made by nonresidents of Weston who are not former residents of Weston must be registered with the city council. Brimley’s mayoral campaign clearly complied with this law since it accepted contributions only from residents and former residents of Weston.

There's a diagram for this question provided below the video, but I often felt like diagramming took more time than I had during some questions on this test -- hence my habit of simplifying the rules. So, how do we simplify this?

Law: All contributions greater than $100 made by nonresidents, who are not former residents, must be registered with the city council.

My next step with these questions is hypotheticals. Hypotheticals show up a lot in law school, and they're important here because they help you test the inferences.

Hypothetical I: Contribution of $1000, made by a former resident of Weston.

Result: Not necessarily registered.

Hypothetical II: Contribution of $50, made by a non-resident and non-former-resident of Weston.

Result: Not necessarily registered.

At this point, I feel pretty confident I understand the law as written. Now we move on to the second portion of the answer choice. We're told that Brimley's campaign clearly complied with this law since it accepted contributions only from residents and former residents of Weston.

So, what does that mean about how Brimley's campaign registered its contributions? Well, the law tells us that campaigns have to report contributions of over $100 from nonresidents who are not former residents. Everything else is off the table -- to use your words, that really does mean he doesn't have to register. This is a place where you have to stick closely to the words on the page. It doesn't make sense that a $1,000,000 contribution from a Weston resident wouldn't have to be registered by a $100.02 contribution from someone in Green Bay, Wisconsin would -- but because the law tells us what has to be registered, we have to stick to that framework.

That's why AC (C) is the correct answer to this question. No contributions to Brimley’s campaign needed to be registered with the city council, because the only situation in which they would need to be registered is if they were over a hundred dollars and donated by someone physically unconnected to the town of Weston.

PrepTests ·
PT101.S3.Q17
User Avatar
7Sage Tutor
Friday, Apr 25

Student Question

Why is D wrong? There are 2 possibilities ( worried or not worried)and yet they did not save money, so they are not cutting back on purchases. The video is not explaining why D is wrong. Just says “ it did not happen”. That is not helpful explanation for a method of reasoning. I was between A and D and need a reason to eliminate D. I saw A and D as correct and of course only one is correct.

Tutor Answer

Let's first quickly review the stimulus:

Widespread layoffs in a region are said to be causing employed people to cut back on spending. But because there has been no unusual increase in the amount of money held in employed people's savings accounts, then it must not be the case that employed people are cutting back on spending.

This bolded portion, the author's argument, asks us to accept an unstated assumption: saving accounts always increase when people cut back on spending. Thus, only if savings accounts decline is it the case that spending has increased. The flip side must also be the case: if they did not decline, then spending must not have decreased. This assumption leads to a binary set of possibilities, in which evidence of one side of the binary disproves the opposite side.

Importantly, it is not the existence of these two sets of possibilities that allows the author to state one must be true. Instead, it is that the author shows that one possibility has not happened. Specifically, the author states that savings accounts have remained unchanged, the corollary of which is that spending has not decreased. Thus, the opposite must not be true: one possibility of the binary has been shown to exist, which precludes the opposite.

But that is not what is said in AC D: Arguing that since two alternative developments exhaust all the plausible possibilities, one of those developments occurred and the other did not. This AC, for all intents and purposes, states that because there exist two possibilities, then one can be said to exist without showing the other does not exist. As discussed above, showing that either the A or B possibility has been ruled out was the crucial second step to the authors argument. This AC, however, leaves out that second step entirely. The upshot is an incomplete argument, which is not what occurred in the stimulus.

PrepTests ·
PT104.S2.P3.Q16
User Avatar
7Sage Tutor
Thursday, May 01

Student Question

I understand that A is wrong because if they are all from the same flock, it’s not representative.

Then why is C wrong? Won’t there be “adult → whole population” error?

I have given it some thought, and I came up with this:

Since it is the “most” question and C can’t deny the validity of the experiment to represent the “adult” group at least, it’s less weakening than B, which can’t be valid for any group of birds since it didn’t control the alternative hypothesis.

Is this the right reasoning? If there’s something better, please let me know.

Also, JY’s reasoning for C was that maybe all he wanted to do was test on adults.

I feel like his reasoning here a posteriori, because it seems like there could be a question where this could be too big assumption compared to other ACs.

Tutor Answer

You bring up a good point regarding C and an "adult--> whole population" error. If the last paragraph had said this experiment provided "definitive proof," then I think you would have been correct in identifying this as something that weakens the claim made in the paragraph.

 

But the last paragraph only mentioned that the experiment provided "further evidence" for the hypothesis in question. That claim is limited, and fits well within the confines of an experiment that only tested adult birds. Evidence that adult birds responded to the "breast stripe" width of dummy birds does support the overall hypothesis, at least as it relates to adult birds. That is significant all on its own, and in turn supports the claim that "further evidence" has been provided.

 

From my perspective, it is this point that JY is making in the video: knowing that "live juvenile birds" were not included in the experiment is irrelevant, because it doesn't weaken the claim that "further evidence" has been provided. In that sense, this may indeed be an example of posteriori knowledge in that JY, from experience, can discern irrelevant information. But I don't think it is due to him having seen this question and/or other similar kinds of questions. Instead, I think it is instead the product of a certain kind of mindset, and an LSAT strategy more broadly: recognizing that there only ever exists one correct answer choice (AC) - all others are fundamentally wrong, not less wrong.

 

This mindset, I believe, helps one be more discerning. In effect, it leads to you "assuming the worst" in an AC: instead of asking yourself why it might be correct, you instead ask why it might be incorrect. I believed this is a helpful bias to have on the LSAT, because it raises the bar for what truly makes an AC relevant. And if it is not relevant, then it must be wrong.

PrepTests ·
PT114.S2.Q12
User Avatar
7Sage Tutor
Thursday, May 01

Student Question

If answer choice B said, “In many surveys, most people DO name high salary as the most desirable feature of a job” would this be a weakener? I think I understand why B is wrong but I want to be clear on exactly why it’s wrong. It’s pretending to attack the premise because it’s trying to say the opposite of what the survey data is saying in the premise. The presence of a new, separate set of survey data that reports the opposite of what another set of survey data reports doesn’t refute or “weaken” the later. We don’t know enough about either set of survey data to determine whether one is superior to another. The two simply came to different conclusions. This explanation, as outlined in the video, makes sense to me. However, by that reasoning, wouldn’t this make answer choice (A) in PT 114.2.12 a correct answer (since it’s an except question)?

Tutor Answer

Your rephrased AC (B) would absolutely weaken this argument. The author's position is that money isn’t the strongest incentive when choosing a job -- the new answer you've provided would completely blow up that logic. Your logic about why the current version of AC (B) doesn't work is also really well executed. Keep thinking about problems this way, and you will go far on this exam.

 

Now, to 114.2.12. This is a very tough Weaken / Except question. I'm sorry the video didn't do what you needed it to do -- let's go through the question together! Because this is an "Except" question, we know that four of these answer choices are going to weaken the argument in the stimulus. One of them won't, and that will be the correct answer. Importantly, just because something doesn't weaken an argument does not mean we can conclude that it strengthens the argument. For example, if I throw eggs at a brick wall, I'm probably not weakening the brick wall -- but I'm certainly not strengthening it. Our correct answer could do nothing, or be totally irrelevant. All we care about is that it doesn't weaken the argument.

 

First, let's go through the incorrect ACs (the four that do weaken the argument):

 

AC (A) weakens the argument by introducing outside evidence that counters the conclusion (that "most people would want to be informed if they had any serious medical condition").

 

AC (B) weakens the argument by directly attacking the survey conducted. If people don't share their true feelings, we have no way to conclude that the survey used as the argument's only point of evidence actually proves... well, anything.

 

AC (D) weakens the argument because it indicates that the survey was bad. If the survey suggested to those taking it that a reasonable person would want to be told if they had a serious medical condition, we can infer that people would be more likely to answer that way -- suggesting that the data used to support the conclusion is faulty at best.

 

AC (E) weakens the argument by significantly biasing the result of the sample. Not only is this new group more likely to be aware of what's happening when the survey is being administered, but you certainly can't support a conclusion about "most people" on the grounds of a survey conducted exclusively of first year psych students.

 

Now, for the correct answer.

 

AC (C) does not weaken the argument. A survey conductor having no background in the field they are conducting a survey on does not mean they would be unable to conduct a good survey. There is a clear and distinct separation between the concept that they are surveying on and the process of conducting the survey itself. Plus, this answer choice says that "some” of the researchers had no background in medicine, which only tells us that this applies to at least one researcher. Other researchers might have.

Confirm action

Are you sure?